Howdy, Stranger!

It looks like you're new here. If you want to get involved, click one of these buttons!

Prep Test 48 LR1 #20

jimophthojimophtho Member
edited September 2015 in Logical Reasoning 16 karma
Researcher: We have found that some cases of high blood pressure can be treated effectively with medicine. Since it is generally accepted that any illness caused by stress is treatable only by the reduction of stress, some cases of high blood pressure must not be caused by stress.

Which one of the following is an assumption required by the researcher's argument?

D) Some conditions that are treated effectively by medicines are not also treatable through the reduction of stress.

E) Medicine used to treat high blood pressure does not itself reduce stress.

The negation of D seems to me to be "All conditions that are effectively treated by medicines are also treatable through the reduction of stress." While the negation of E is that "Medicine used to treat high blood pressure does itself reduce stress."

In my mind, both of these seem to break the logic of the argument. I understand that E is correct because, having diagrammed it, I understand that a some statement will not suffice and that I need a conditional statement to bridge the gap between the premises and the conclusion. However, the negation test did not allow me to eliminate D, but it is supposedly infallible in determining the correct answer. I am a bit concerned that both D and E seem for me to break the argument. How is it that D does not do that? Or if it does, how do I know to trust the negation test? Is it that the negation of D says that medicine and stress reduction are both separately effective and that E says that the medicine is effective because it reduces the stress? Is the negation of D even applicable, since it would contradict the premise that states that an illness caused by stress can only be treated by a reduction of stress? Thanks

Comments

  • inactiveinactive Alum Member
    12637 karma
    Bumping this thread to the top so more people can see it.
  • c.janson35c.janson35 Free Trial Inactive Sage Inactive ⭐
    edited September 2015 2398 karma
    D says: "Some conditions that are treated effectively by medicines are not also treatable through the reduction of stress."

    The reason why this isn't a necessary assumption is because the conclusion is about conditions treated by the reduction of stress, so we don't care about the conditions that are treated by medicine not being able to be treatable through the reduction of stress. What we care about, and what the premises/conclusion are/is about, are those conditions that are only treatable through the reduction of stress. Because HBP can be treated by something other than a reduction in stress, it seems logical to conclude that some HBP must not be caused by stress. But, it must be true that the meds that work to treat HBP don't themselves reduce stress, because then we would have no support for the conclusion.

    Hope this helps!
Sign In or Register to comment.